Регистрирайте сеРегистрирайте се

Екстремални Задачи и Решения

Иди на страница Предишна  1, 2, 3, 4, 5, 6, 7, 8  Следваща
 
   Форум за математика Форуми -> Олимпиади и състезания за 9-12 клас
Предишната тема :: Следващата тема  
Автор Съобщение
Titu_Andrescu
Напреднал


Регистриран на: 28 Oct 2006
Мнения: 370

Репутация: 68.9
гласове: 29

МнениеПуснато на: Sat Dec 22, 2007 5:17 pm    Заглавие:

Хубаво решение!

Може да подходим и по следния начин на V. Cirtoaje.
Решение.
Неравенството е еквивалентно на [tex]\frac{a}{b+c}+\frac{b}{c+a}+\frac{c}{a+b}-\frac32\ge \frac23\left(1-\frac{ab+bc+ca}{a^2+b^2+c^2}\right)[/tex].
Тъй като [tex]\sum_{cyc}\left(\frac{a}{b+c}-\frac{1}{2}\right)=\sum_{cyc}\frac{(a-b)+(a-c)}{2(b+c)}=\sum_{cyc}\frac{a-b}{2(b+c)}+\sum_{cyc}\frac{b-a}{2(c+a)}=\sum_{cyc}\left(\frac{a-b}{2}\left(\frac{1}{b+c}-\frac{1}{c+a}\right)\right)=\sum_{cyc}\frac{(a-b)^2}{2(b+c)(c+a)}[/tex]

и

[tex]\frac23\left(1-\frac{ab+bc+ca}{a^2+b^2+c^2}\right)=\sum_{cyc}\frac{(a-b)^2}{3(a^2+b^2+c^2)}[/tex],

то неравенството става еквивалентно на

[tex]\sum_{cyc}(a-b)^2\left(\frac{1}{2(b+c)(c+a)}-\frac{1}{3(a^2+b^2+c^2)}\right)\ge 0[/tex], което е вярно, защото

[tex]3(a^2+b^2+c^2)-2(b+c)(c+a)=(a+b-c)^2+2(a-b)^2\ge 0[/tex].

С това задачата е решета. Равенство се достига [tex]\Leftrightarrow a=b=c[/tex].
Върнете се в началото
Вижте профила на потребителя Изпратете лично съобщение
Реклама







Пуснато на:     Заглавие: Реклама

Върнете се в началото
Мирослав Стоенчев
Напреднал


Регистриран на: 21 Aug 2007
Мнения: 279

Репутация: 72
гласове: 45

МнениеПуснато на: Sat Dec 22, 2007 11:51 pm    Заглавие:

Към задача 48 съм пропуснал да запиша доказателство на неравенството:

[tex]3(x^{2}+y^{2}+z^{2})\left\[2x^{2}+(y+z)^{2}\right\]>(x+y)(x+z)(x+y+z)(2x+y+z),[/tex]

при предположението [tex]x,y,z\in \R^{+};\min\left\{x,y,z\right\}=x.[/tex]

Д-во: Понеже неравенството е хомогенно относно променливите [tex]x,y,z,[/tex] то б.о.о. ще считаме, че [tex]x+y+z=1.[/tex] Toгава [tex]3x\le x+y+z=1\Rightarrow x\le \frac{1}{3}.[/tex] Използвайки

[tex]x^{2}+y^{2}+z^{2}=x^{2}+(y+z)^{2}-2yz=x^{2}+(1-x)^{2}-2yz=2x^{2}-2x+1-2yz; 2x^{2}+(y+z)^{2}=2x^{2}+(1-x)^{2};(x+y)(x+z)=x^{2}+(y+z)x+yz=x^{2}+(1-x)x+yz=x+yz,[/tex]

можем да преобразуваме изходното неравенство във вида:

[tex]3(2x^{2}-2x+1-2yz)(3x^{2}-2x+1)>(x+yz)(1+x)[/tex]

Сега съобразяваме, че [tex]yz\le \left\(\frac{y+z}{2}\right\)^{2}=\left\(\frac{1-x}{2}\right\)^{2}\Rightarrow 3(2x^{2}-2x+1-2yz)(3x^{2}-2x+1)\ge 3\left\[2x^{2}-2x+1-2\left\(\frac{1-x}{2}\right\)^{2}\right\](3x^{2}-2x+1); (x+yz)(1+x)\le \left\[x+\left\(\frac{1-x}{2}\right\)^{2}\right\](1+x)[/tex]

Достатъчно е да докажем, че [tex]3\left\[2x^{2}-2x+1-2\left\(\frac{1-x}{2}\right\)^{2}\right\](3x^{2}-2x+1)>\left\[x+\left\(\frac{1-x}{2}\right\)^{2}\right\](1+x),[/tex]
при [tex]x\le \frac{1}{3}.[/tex] Неравенството е еквивалентно на

[tex]F(x)=54x^{4}-73x^{3}+57x^{2}-27x+5>0,[/tex] при [tex]0\le x\le\frac{1}{3}.[/tex] Лесно се показва, че [tex]F(x)>0,[/tex] при [tex]x\in \left\[0,\frac{2}{5}\right\]\Rightarrow \min_{0\le x\le\frac{1}{3}}F(x)>0.[/tex] С това доказателството е завършено.
Върнете се в началото
Вижте профила на потребителя Изпратете лично съобщение
sd_pld
Начинаещ


Регистриран на: 05 Dec 2006
Мнения: 68

Репутация: 12.9
гласове: 1

МнениеПуснато на: Mon Dec 24, 2007 3:34 pm    Заглавие:

Titu_Andrescu написа:



Сега, от Тв.1 и от неравенството на Коши-Буняковски-Шварц следва, че
[tex]\frac{h_a}{m_b}+\frac{h_b}{m_c}+\frac{h_c}{m_a}\le 3[/tex].


Мисля ,че за да се стигне до това просто може да се приложи
[tex] h_{i} \le m_{i} [/tex] ,което е валидно не само за медианите Smile
Върнете се в началото
Вижте профила на потребителя Изпратете лично съобщение
Titu_Andrescu
Напреднал


Регистриран на: 28 Oct 2006
Мнения: 370

Репутация: 68.9
гласове: 29

МнениеПуснато на: Mon Dec 24, 2007 4:56 pm    Заглавие:

и така ще докажеш, че [tex]\frac{h_a}{m_a}+\frac{h_b}{m_b}+\frac{h_c}{m_c}\le 3[/tex], а неравенството което сме доказали е друго
Върнете се в началото
Вижте профила на потребителя Изпратете лично съобщение
sd_pld
Начинаещ


Регистриран на: 05 Dec 2006
Мнения: 68

Репутация: 12.9
гласове: 1

МнениеПуснато на: Mon Dec 24, 2007 9:01 pm    Заглавие:

Titu_Andrescu написа:
и така ще докажеш, че [tex]\frac{h_a}{m_a}+\frac{h_b}{m_b}+\frac{h_c}{m_c}\le 3[/tex], а неравенството което сме доказали е друго

Извинявам се ,грешката е моя.
Върнете се в началото
Вижте профила на потребителя Изпратете лично съобщение
Мирослав Стоенчев
Напреднал


Регистриран на: 21 Aug 2007
Мнения: 279

Репутация: 72
гласове: 45

МнениеПуснато на: Tue Dec 25, 2007 3:54 pm    Заглавие:

Задача 46. Д-во: Неравенството е еквивалентно на [tex]\sum_{cyclic}\frac{x^{2}+y^{2}+z^{2}}{x^{5}+y^{2}+z^{2}}\le 3.[/tex] Полагаме [tex]a=x^{2};b=y^{2};c=z^{2}; a=ka_{1},b=kb_{1},c=kc_{1}; a_{1}+b_{1}+c_{1}=3\Rightarrow k\ge 1.[/tex] Тогава [tex]\sum_{c}\frac{x^{2}+y^{2}+z^{2}}{x^{5}+y^{2}+z^{2}}=\sum_{c}\frac{a_{1}+b_{1}+c_{1}}{k^{\frac{3}{2}}(a_{1})^{\frac{5}{2}}+b_{1}+c_{1}}\le \sum_{c}\frac{a_{1}+b_{1}+c_{1}}{(a_{1})^{\frac{5}{2}}+b_{1}+c_{1}}=\sum_{c}\frac{3}{(a_{1})^{\frac{5}{2}}+b_{1}+c_{1}}\le 3\Leftrightarrow \sum_{c}\frac{1}{(a_{1})^{\frac{5}{2}}+b_{1}+c_{1}}\le 1.[/tex] Достатъчно е да докажем, че

[tex]\sum_{c}\frac{1}{(a_{1})^{\frac{5}{2}}+b_{1}+c_{1}}\le 1,[/tex] при [tex]a_{1}+b_{1}+c_{1}=3.[/tex]

Ще използваме следната

Теорема.(Vasile Cirtoaje) Нека [tex]x_{1},x_{2},...,x_{n}[/tex] са неотрицателни реални числа, за които [tex]x_{1}+x_{2}+...+x_{n}=n,[/tex] и [tex]p>1,p\in \R.[/tex]
Тогава [tex]\sum_{c}\frac{1}{x_{1}^{p}+x_{2}+...+x_{n}}\le 1.[/tex]

Прилагаме теоремата Cirtoaje за [tex]n=3,p=\frac{5}{2}\Rightarrow \sum_{c}\frac{1}{(a_{1})^{\frac{5}{2}}+b_{1}+c_{1}}\le 1.[/tex] С това твърдението е доказано.
Върнете се в началото
Вижте профила на потребителя Изпратете лично съобщение
Titu_Andrescu
Напреднал


Регистриран на: 28 Oct 2006
Мнения: 370

Репутация: 68.9
гласове: 29

МнениеПуснато на: Tue Dec 25, 2007 5:13 pm    Заглавие:

Задача 49.
а) Да се намери най-малкото положително число k, за което неравенството
[tex]\frac{1}{ab}+\frac{1}{bc}+\frac{1}{ca}\le \frac{k}{S}[/tex] е изпълнено за всеки триъгълник със страни [tex]a, b, c[/tex] и лице [tex]S[/tex].
б) Съществува ли положително число [tex]K[/tex], за което неравенството
[tex]\frac{1}{a^2}+\frac{1}{b^2}+\frac{1}{c^2}\le \frac{K}{S}[/tex] е изпълнено за всеки триъгълник със страни [tex]a, b, c[/tex] и лице [tex]S[/tex].
(Камен Алексиев, (kamen05))
Върнете се в началото
Вижте профила на потребителя Изпратете лично съобщение
Мирослав Стоенчев
Напреднал


Регистриран на: 21 Aug 2007
Мнения: 279

Репутация: 72
гласове: 45

МнениеПуснато на: Wed Dec 26, 2007 12:31 am    Заглавие:

Задача 47. Д-во: Б.о.о. ще считаме, че [tex]c=1.[/tex] Наистина, равенството и неравенството зададени в условието са хомогенни, следователно можем да извършим смяната [tex](a,b,c)\rightarrow(\frac{a}{c},\frac{b}{c},1).[/tex]

Имаме [tex](a+b-1)(b+a-ab)=4ab\Leftrightarrow (a+b-1)(\frac{a+b}{ab}-1)=4.[/tex] Полагаме [tex]x=a+b;y=\frac{a+b}{ab}\Rightarrow (x-1)(y-1)=4\Leftrightarrow x+y=xy-3;x,y\in \R^{+};ab=\frac{x}{y}.[/tex]
Също така [tex]2\sqrt{xy}\le x+y=xy-3\Rightarrow xy-2\sqrt{xy}-3\ge 0\Rightarrow t=xy\ge 9.[/tex]

Нека [tex]H(a,b)=(a^{4}+b^{4}+1)(a^{-4}+b^{-4}+1).[/tex] Пресмятаме [tex]a^{4}+b^{4}=\left\[(a+b)^{2}-2ab\right\]^{2}-2(ab)^{2}=(x^{2}-\frac{2x}{y})^{2}-2\left\(\frac{x}{y}\right\)^{2}=x^{4}-4\frac{x^{3}}{y}+2\left\(\frac{x}{y}\right\)^{2}.[/tex] Сега лесно намираме [tex]H(a,b)=F(x,y,t)=\left\(x^{4}-4\frac{x^{3}}{y}+2\left\(\frac{x}{y}\right\)^{2}+1\right\)\left\(\frac{x^{4}-4\frac{x^{3}}{y}+2\left\(\frac{x}{y}\right\)^{2}}{\left\(\frac{x}{y}\right\)^{4}}+1\right\)=\left\(x^{4}-4\frac{x^{3}}{y}+2\left\(\frac{x}{y}\right\)^{2}+1\right\)\left\(y^{4}-4\frac{y^{3}}{x}+2\left\(\frac{y}{x}\right\)^{2}+1\right\)=t^{4}-8t^{3}+20t^{2}-16t+5+(x^{4}+y^{4})(1-\frac{4}{t}+\frac{2}{t^{2}}).[/tex]

Знаем, че [tex]t=xy\ge 9\Rightarrow 1-\frac{4}{t}+\frac{2}{t^{2}}>0\Rightarrow (x^{4}+y^{4})(1-\frac{4}{t}+\frac{2}{t^{2}})\ge 2(xy)^{2}(1-\frac{4}{t}+\frac{2}{t^{2}})=2t^{2}-8t+4.[/tex] Тогава

[tex]H(a,b)=F(x,y,t)\ge t^{4}-8t^{3}+22t^{2}-24t+9=G(t); t\ge 9.[/tex]

[tex]G'(t)=4(t-1)(t-2)(t-3)>0, t>3 \Rightarrow G(t)[/tex] е строго растяща при [tex]t\in (3,+\infty)\Rightarrow \min_{t\ge 9}G(t)=G(9)=2304.[/tex] Taка доказахме, че [tex]H(a,b)\ge G(t)\ge \min_{t\ge 9}G(t)=G(9)=2304.[/tex]
Върнете се в началото
Вижте профила на потребителя Изпратете лично съобщение
Мирослав Стоенчев
Напреднал


Регистриран на: 21 Aug 2007
Мнения: 279

Репутация: 72
гласове: 45

МнениеПуснато на: Wed Dec 26, 2007 10:28 am    Заглавие:

Задача 49. а) Знаем, че [tex]S_{\triangle}=\frac{ab.\sin(\alpha)}{2}=\frac{bc.\sin(\beta)}{2}=\frac{ca.\sin(\gamma)}{2};\sin(\alpha)+\sin(\beta)+\sin(\gamma)\le \frac{3\sqrt{3}}{2}.[/tex] Toгава

[tex]k\ge \frac{S}{ab}+\frac{S}{bc}+\frac{S}{ca}=\frac{1}{2}(\sin(\alpha)+\sin(\beta)+\sin(\gamma))\Rightarrow k=\max \frac{1}{2}(\sin(\alpha)+\sin(\beta)+\sin(\gamma))=\frac{3\sqrt{3}}{4}.[/tex]

б) Ще докажем, че не съществува положителна константа [tex]K,[/tex] за която [tex]\frac{1}{a^{2}}+\frac{1}{b^{2}}+\frac{1}{c^{2}}\le \frac{K}{S}[/tex]

Нека [tex]n\in N,[/tex] да разгледаме [tex]\triangle : h_{a}=2n.a\Rightarrow K\ge \frac{S}{a^{2}}+\frac{S}{b^{2}}+\frac{S}{c^{2}}>\frac{S}{a^{2}}=\frac{a.h_{a}}{2a^{2}}=n.[/tex]
Така доказахме, че за произволно естествено [tex]n[/tex] трябва да бъде в сила [tex]K>n.[/tex] Следователно не съществува константа със свойството от б).
Върнете се в началото
Вижте профила на потребителя Изпратете лично съобщение
Мирослав Стоенчев
Напреднал


Регистриран на: 21 Aug 2007
Мнения: 279

Репутация: 72
гласове: 45

МнениеПуснато на: Thu Dec 27, 2007 3:39 pm    Заглавие:

Задача 4. (Иранското неравенство-второ решение). Д-во: Трябва да докажем, че при положителни [tex]x,y,z\Rightarrow (xy+yz+zx)\left\(\frac{1}{(x+y)^{2}}+\frac{1}{(y+z)^{2}}+\frac{1}{(z+x)^{2}}\right\)\ge \frac{9}{4}.[/tex]

Полагаме [tex]G(x,y,z)=(xy+yz+zx)\left\(\frac{1}{(x+y)^{2}}+\frac{1}{(y+z)^{2}}+\frac{1}{(z+x)^{2}}\right\);[/tex]

[tex]\frac{y}{x}=t\in R^{+}\Rightarrow G(x,y,y)=(2xy+y^{2})\left\(\frac{2}{(x+y)^{2}}+\frac{1}{4y^{2}}\right\)=\frac{(2+t)(9t^{2}+2t+1)}{4t(1+t)^{2}}=h(t).[/tex] Тогава [tex]h(t)\ge \frac{9}{4}\Leftrightarrow 2(t-1)^{2}\ge 0.[/tex] Така доказахме, че [tex]G(x,y,y)\ge \min_{y>0}G(x,y,y)=\min_{t>0}h(t)=h(1)=\frac{9}{4}.[/tex]

Нека [tex]\min\left\{x,y,z\right\}=x.[/tex] Тогава пресмятаме разликата:

[tex]G(x,y,z)-G(x,\frac{y+z}{2},\frac{y+z}{2})=\frac{(y-z)^{2}\left\{4(y+z)^{2}(xy+yz+zx)\left\[(4x+y+z)(2x+y+z)+2(yz-x^{2})\right\]-(x+y)^{2}(z+x)^{2}\left\[8(y+z)^{2}+(2x+y+z)^{2}\right\]\right\}}{4(x+y)^{2}(y+z)^{2}(z+x)^{2}(2x+y+z)^{2}}\ge 0,[/tex]
понеже

[tex]4(y+z)^{2}(xy+yz+zx)\left\[(4x+y+z)(2x+y+z)+2(yz-x^{2})\right\]-(x+y)^{2}(z+x)^{2}\left\[8(y+z)^{2}+(2x+y+z)^{2}\right\]>0,[/tex] при положителни [tex]x,y,z[/tex] за които [tex]\min\left\{x,y,z\right\}=x.[/tex]

Следователно [tex]G(x,y,z)-G(x,\frac{y+z}{2},\frac{y+z}{2})\ge 0\Rightarrow G(x,y,z)\ge G(x,\frac{y+z}{2},\frac{y+z}{2})\ge \min_{y>0}G(x,y,y)=\frac{9}{4}.[/tex] Доказателството е завършено.
Върнете се в началото
Вижте профила на потребителя Изпратете лично съобщение
Titu_Andrescu
Напреднал


Регистриран на: 28 Oct 2006
Мнения: 370

Репутация: 68.9
гласове: 29

МнениеПуснато на: Thu Dec 27, 2007 5:37 pm    Заглавие:

Красота!
Върнете се в началото
Вижте профила на потребителя Изпратете лично съобщение
krassi_holmz
Редовен


Регистриран на: 05 Jan 2006
Мнения: 146
Местожителство: Ню Йорк, BG
Репутация: 57.9
гласове: 18

МнениеПуснато на: Thu Dec 27, 2007 8:30 pm    Заглавие:

Метода на Стоенчев от иранското неравенство е много удобен.
Нарича се аритметична компенсация и с него могат да се докажат доста остри неравенства като иранското.
На мен също ми харесва второто решение, но аз лично предпочитам Schur/Muirhead и повечко "акробатика".

Ето малко повече информация за аритметичната компенсация:



2006_3_arithmeticcompensation.pdf
 Description:

Свали
 Име на файл:  2006_3_arithmeticcompensation.pdf
 Големина на файла:  87.61 KB
 Свален:  1524 пъти(s)

Върнете се в началото
Вижте профила на потребителя Изпратете лично съобщение
Titu_Andrescu
Напреднал


Регистриран на: 28 Oct 2006
Мнения: 370

Репутация: 68.9
гласове: 29

МнениеПуснато на: Fri Dec 28, 2007 2:05 pm    Заглавие:

Задача 50. При стандартните означения за един триъгълник да се докаже, че [tex]\frac{m_am_b}{ab}+\frac{m_bm_c}{bc}+\frac{m_cm_a}{ca}\ge \frac94[/tex].

(Някой виждал ли е тази задача преди? Измислих я докато решавах една друга, дори открих едно найстина много интересно доказателство,
но не знам дали някой не е открил тази зависимост преди мен, защото изглежда доста добре)

Задача 51. Едно от двете неравенства [tex](sinx)^{sinx}<(cosx)^{cosx}[/tex] и [tex](sinx)^{sinx}>(cosx)^{cosx}[/tex]
е винаги вярно за всички реални числа [tex]x[/tex], за които [tex]0<x<\frac{\pi}{4}[/tex]. Откриите кое е то и го докажете.
(Предложена от Weichao Wu, задачата е побликувана и в книгата на Д-р. Титу Андреску и Zuming Feng - “103 Trigonometry Problems From The Training Of The USA IMO Team“)



103 Trigonometry Problems.jpg
 Description:
 Големина на файла:  10.41 KB
 Видяна:  4199 пъти(s)

103 Trigonometry Problems.jpg


Върнете се в началото
Вижте профила на потребителя Изпратете лично съобщение
LogiC
Начинаещ


Регистриран на: 28 Dec 2007
Мнения: 3
Местожителство: Sofia / Simeonovo
Репутация: 1.3
гласове: 1

МнениеПуснато на: Fri Dec 28, 2007 9:42 pm    Заглавие:

Привет ! Нов съм тук и малко не съм в час с нещата за това ще Ви помоля да ми отговорите на няколко въпроса :
1) От каде са по-горе решаваните задачи(сборник,състезание,олимпиада)
2) Ако можете да ми дадете линк с материали на тема "Екстремални задачи" и в частност ако има в геометрията Smile

Благодаря предварително, надявам се, че ще можете да ми помогнете .
Върнете се в началото
Вижте профила на потребителя Изпратете лично съобщение Посетете сайта на потребителя
Titu_Andrescu
Напреднал


Регистриран на: 28 Oct 2006
Мнения: 370

Репутация: 68.9
гласове: 29

МнениеПуснато на: Fri Dec 28, 2007 11:37 pm    Заглавие:

След всяка задача има информация от къде идва, а ако няма - значи е фолклорна т.е. e резултат на много математици.
Материал по тази тема може да си набавиш от форума.
Върнете се в началото
Вижте профила на потребителя Изпратете лично съобщение
Мирослав Стоенчев
Напреднал


Регистриран на: 21 Aug 2007
Мнения: 279

Репутация: 72
гласове: 45

МнениеПуснато на: Sat Dec 29, 2007 12:57 am    Заглавие:

Задача 1. б) Трябва да докажем, че [tex]\frac{1}{5-2ab}+\frac{1}{5-2bc}+\frac{1}{5-2ca}\le 1,[/tex] при неотрицателни [tex]a,b,c[/tex] за които [tex] a^{2}+b^{2}+c^{2}=3.[/tex]

Д-во: Полагаме [tex]g(x,y,z)=\frac{1}{5-2xy}+\frac{1}{5-2yz}+\frac{1}{5-2zx},[/tex] [tex]x^{2}+y^{2}+z^{2}=3.[/tex]

1сл. [tex]y=z=0\Rightarrow g(x,0,0)=\frac{3}{5}<1.[/tex]

2сл. [tex]z=0\Rightarrow g(x,y,0)=\frac{1}{5-2xy}+\frac{2}{5}\le \frac{1}{5-(x^{2}+y^{2})}+\frac{2}{5}=\frac{1}{5-3}+\frac{2}{5}=\frac{1}{2}+\frac{2}{5}=\frac{9}{10}<1.[/tex]

3сл. [tex]\min\left\{x,y,z\right\}>0.[/tex] Пресмятаме [tex]g(x,y,y)=\frac{2}{5-2xy}+\frac{1}{5-2y^{2}},[/tex] при [tex]x^{2}+2y^{2}=3\Leftrightarrow y=\sqrt{\frac{3-x^{2}}{2}}.[/tex]

Toгава [tex]g(x,y,y)\le 1\Leftrightarrow y\le \frac{1+3x^{2}}{2x(1+x^{2})}\Leftrightarrow \sqrt{\frac{3-x^{2}}{2}}\le \frac{1+3x^{2}}{2x(1+x^{2})}\Leftrightarrow (x^{2}-1)^{2}(2x^{4}+2x^{2}+1)\ge 0.[/tex] Така доказахме, че [tex]g(x,y,y)\le 1.[/tex]

Нека [tex]\min\left\{x,y,z\right\}=x.[/tex] Тогава [tex] 3x^{2}\le x^{2}+y^{2}+z^{2}=3\Rightarrow x\le 1.[/tex] Полагаме [tex]k=\frac{y^{2}+z^{2}}{2}[/tex] и пресмятаме разликата

[tex]g(x,\sqrt{\frac{y^{2}+z^{2}}{2}},\sqrt{\frac{y^{2}+z^{2}}{2}})-g(x,y,z)=\frac{(y-z)^{2}\left\{(5-2xz)(5-2xy)(5-2x\sqrt{k})(y+\sqrt{k})(z+\sqrt{k})+x(x^{2}+2)(y+z)(5-2yz)\left\[5-2x(\sqrt{k}+y+z)\right\]\right\}}{(5-2xy)(5-2yz)(5-2zx)(x^{2}+2)(5-2x\sqrt{k})(y+\sqrt{k})(z+\sqrt{k})}\ge 0,[/tex]
понеже
[tex](5-2xz)(5-2xy)(5-2x\sqrt{k})(y+\sqrt{k})(z+\sqrt{k})+x(x^{2}+2)(y+z)(5-2yz)\left\[5-2x(\sqrt{k}+y+z)\right\]>0,[/tex] при положителни [tex]x,y,z[/tex] за които [tex]\min\left\{x,y,z\right\}=x[/tex] и [tex]x^{2}+y^{2}+z^{2}=3.[/tex]
Така доказахме, че [tex]g(x,y,z)\le g(x,\sqrt{k},\sqrt{k})\le \max_{x^{2}+2y^{2}=3}g(x,y,y)=1. [/tex] Твърдението е доказано.
Върнете се в началото
Вижте профила на потребителя Изпратете лично съобщение
Titu_Andrescu
Напреднал


Регистриран на: 28 Oct 2006
Мнения: 370

Репутация: 68.9
гласове: 29

МнениеПуснато на: Sat Dec 29, 2007 11:14 am    Заглавие:

Задача 44. Решение.
Лема. За две положителни реални числа [tex]x,y[/tex] е изпълнено неравенството [tex]\sqrt[3]{\frac{x^3+y^3}{2}}\le\frac{x^2+y^2}{x+y}[/tex].
Д-во: Повдигаме и двете страни на неравенството на степен [tex]3[/tex] и виждаме, че е еквивалентно на [tex](x-y)^4(x^2+xy+y^2)\ge 0[/tex].

Сега, от лемата следва, че [tex]\sum_{cyc}\sqrt[3]{\frac{a^3+b^3}{2}}\le\sum_{cyc}\frac{a^2+b^2}{a+b}[/tex], но [tex]a+b-\frac{a^2+b^2}{a+b}=\frac{2ab}{a+b}=\frac{2}{\frac{1}{a}+\frac{1}{b}}[/tex].
Следователно неравенството което искаме да докажем е еквивалентно на [tex]\sum_{cyc}\frac{1}{\frac{1}{a}+\frac{1}{b}}\ge 2[/tex].

Но от неравенството на Коши-Буняковски-Шварц следва, че [tex]\sum_{cyc}\frac{1}{\frac{1}{a}+\frac{1}{b}}\ge \frac{4^2}{2\left(\frac{1}{a}+\frac{1}{b}+\frac{1}{c}+\frac{1}{d}\right)}=\frac{16}{8}=2[/tex], което искахме да докажем.
Равенство се достига [tex]\Leftrightarrow a=b=c=d[/tex].
Върнете се в началото
Вижте профила на потребителя Изпратете лично съобщение
Мирослав Стоенчев
Напреднал


Регистриран на: 21 Aug 2007
Мнения: 279

Репутация: 72
гласове: 45

МнениеПуснато на: Sat Dec 29, 2007 11:23 am    Заглавие:

Задача 1. б) Ще докажем, че при положителни [tex]x,y,z[/tex] за които [tex]\min\left\{x,y,z\right\}=x[/tex] и [tex]x^{2}+y^{2}+z^{2}=3,[/tex] е в сила неравенството

[tex](5-2xz)(5-2xy)(5-2x\sqrt{k})(y+\sqrt{k})(z+\sqrt{k})+x(x^{2}+2)(y+z)(5-2yz)\left\[5-2x(\sqrt{k}+y+z)\right\]>0.[/tex]

Да забележим, че всички множители участващи в неравенството са положителни, евентуално с изключение на един и той е [tex]5-2x(\sqrt{k}+y+z).[/tex]

1сл. [tex]5-2x(\sqrt{k}+y+z)\ge 0\Rightarrow [/tex] неравенството е вярно.

2сл. [tex]5-2x(\sqrt{k}+y+z)<0. [/tex] Имаме

[tex]i)[/tex] [tex]x^{2}+2=5-(y^{2}+z^{2})\le 5-2yz\le \min(5-2xy,5-2xz)\Rightarrow (x^{2}+2)(5-2yz)\le (5-2yz)^{2}\le (5-2xz)(5-2xy).[/tex]

[tex]ii)[/tex] [tex]x(y+z)=xy+xz\le 2yz\le yz+k<(y+\sqrt{k})(z+\sqrt{k})[/tex]

[tex]iii)[/tex] [tex]-\left\(5-2x(\sqrt{k}+y+z)\right\)<5-2x\sqrt{k}\Leftrightarrow x(2\sqrt{k}+y+z)<5.[/tex] Но [tex]x(2\sqrt{k}+y+z)\le 1(2\sqrt{k}+2\sqrt{\frac{y^{2}+z^{2}}{2}})=4\sqrt{k}<4\sqrt{\frac{3}{2}}<5.[/tex]

Като вземем в предвид [tex]i);ii);iii)[/tex] заключаваме, че [tex](5-2xz)(5-2xy)(5-2x\sqrt{k})(y+\sqrt{k})(z+\sqrt{k})+x(x^{2}+2)(y+z)(5-2yz)\left\[5-2x(\sqrt{k}+y+z)\right\]>0.[/tex]
Твърдението е доказано.
Върнете се в началото
Вижте профила на потребителя Изпратете лично съобщение
Мирослав Стоенчев
Напреднал


Регистриран на: 21 Aug 2007
Мнения: 279

Репутация: 72
гласове: 45

МнениеПуснато на: Sat Dec 29, 2007 5:36 pm    Заглавие:

Задача 4. Нека [tex]x,y,z\in R^{+}[/tex] и [tex]\min\left\{x,y,z\right\}=x.[/tex] Ще докажем,че [tex]4(y+z)^{2}(xy+yz+zx)\left\[(4x+y+z)(2x+y+z)+2(yz-x^{2})\right\]-(x+y)^{2}(y+z)^{2}\left\[8(y+z)^{2}+(2x+y+z)^{2}\right\]>0.[/tex]

Понеже неравенството е хомогенно относно [tex]x,y,z[/tex] то б.о.о. ще считаме, че [tex]x+y+z=1.[/tex] Тогава [tex]3x\le x+y+z=1\Rightarrow x\le \frac{1}{3}.[/tex] Полагаме
[tex]yz=t\Rightarrow 0<t=yz\le \left\(\frac{y+z}{2}\right\)^{2}=\left\(\frac{1-x}{2}\right\)^{2}.[/tex] Преобразуваме изходното неравенство по начина от задача 48, и стигаме до еквивалентното му неравенство:

[tex]H(x,t)=-(x+1)^{2}t^{2}+\left\[4(1-x)^{2}(-x^{2}+2x+1)-2x(1+x)^{2}\right\]t+G(x)>0.[/tex] Имаме [tex]\min_{0<x\le \frac{1}{3},0<t\le \left\(\frac{1-x}{2}\right\)^{2}}H(x,t)=\min_{0<x\le \frac{1}{3}}\left\{H(x,0),H(x,\left\(\frac{1-x}{2}\right\)^{2})\right\}.[/tex] Достатъчно е да докажем, че [tex]H(x,0)>0;H(x,\left\(\frac{1-x}{2}\right\)^{2})>0.[/tex] Пресмятаме

[tex]H(x,0)=G(x)=x\left\{4(1-x)^{2}\left\[(1-x)(x^{2}+4x+1)-2x\right\]-x(x+1)^{2}\right\}.[/tex]

Полагаме [tex]u=1-x\Rightarrow \frac{2}{3}\le u<1\Rightarrow \frac{G(x)}{x}=h(u)=4u^{5}-24u^{4}+33u^{3}-13u^{2}+8u-4>0,[/tex] понеже [tex]h(u)[/tex] е растяща в интервала [tex]\left\(\frac{2}{3},1\right\)\Rightarrow \min_{\frac{2}{3}\le u<1}h(u)=h(\frac{2}{3})>0.[/tex] Така доказахме, че [tex]H(x,0)=G(x)>0.[/tex] Сега пресмятаме

[tex]H(x,\left\(\frac{1-x}{2}\right\)^{2})=H(1-u,\frac{u^{2}}{4})=-\frac{(2u-u^{2})^{2}}{4}+(-2u^{5}+u^{4}-u^{3}+8u^{2}-4u)+(1-u)G(1-u)\ge -2u^{5}+u^{4}-u^{3}+8u^{2}-4u-\frac{1}{4}=s(u).[/tex] Лесно се показва, че [tex]s(u)[/tex] е растяща в интервала [tex](\frac{2}{3},1)\Rightarrow \min_{\frac{2}{3}\le u<1}s(u)=s(\frac{2}{3})>0,[/tex] т.е. [tex]H(x,\left\(\frac{1-x}{2}\right\)^{2})>0.[/tex] Твърдението е доказано.
Върнете се в началото
Вижте профила на потребителя Изпратете лично съобщение
Titu_Andrescu
Напреднал


Регистриран на: 28 Oct 2006
Мнения: 370

Репутация: 68.9
гласове: 29

МнениеПуснато на: Sun Dec 30, 2007 1:12 pm    Заглавие:

Задача 52. Нека a,b,c са положителни реални числа. Да се докаже, че [tex]\frac{a}{\sqrt{a^2+8bc}}+\frac{b}{\sqrt{b^2+8ca}}+\frac{c}{\sqrt{c^2+8ab}}\ge 1[/tex].
(IMO 2001 г.)



IMO2001.jpg
 Description:
 Големина на файла:  16.5 KB
 Видяна:  4050 пъти(s)

IMO2001.jpg


Върнете се в началото
Вижте профила на потребителя Изпратете лично съобщение
Мирослав Стоенчев
Напреднал


Регистриран на: 21 Aug 2007
Мнения: 279

Репутация: 72
гласове: 45

МнениеПуснато на: Sun Dec 30, 2007 4:32 pm    Заглавие:

Задача 52. Полагаме [tex]x=\frac{8bc}{a^{2}};y=\frac{8ca}{b^{2}};z=\frac{8ab}{c^{2}}\Rightarrow xyz=512.[/tex] Трябва да докажем, че [tex]\frac{1}{\sqrt{1+x}}+\frac{1}{\sqrt{1+y}}+\frac{1}{\sqrt{1+z}}\ge 1.[/tex] Последното е еквивалентно на

[tex]\sum_{c}(1+x)(1+y)+2\sqrt{(1+x)(1+y)(1+z)}\sum_{c}\sqrt{1+x}\ge (1+x)(1+y)(1+z)\Leftrightarrow[/tex]

[tex] 2\sqrt{(1+x)(1+y)(1+z)}\sum_{c}\sqrt{1+x}\ge -2-(x+y+z)+xyz\Leftrightarrow (x+y+z)+2\sqrt{(1+x)(1+y)(1+z)}\sum_{c}\sqrt{1+x}\ge 510.[/tex] Но

[tex]x+y+z\ge 3\sqrt[3]{xyz}=3.8=24; xy+yz+zx\ge 3\sqrt[3]{(xyz)^{2}}=192\Rightarrow (1+x)(1+y)(1+z)=1+(x+y+z)+(xy+yz+zx)+xyz\ge 1+24+192+512=729=3^{6}.[/tex]
Следователно
[tex]\sum_{c}\sqrt{1+x}=\sqrt{1+x}+\sqrt{1+y}+\sqrt{1+z}\ge 3\sqrt[6]{(1+x)(1+y)(1+z)}\ge 9.[/tex] Окончателно

[tex](x+y+z)+2\sqrt{(1+x)(1+y)(1+z)}(\sqrt{1+x}+\sqrt{1+y}+\sqrt{1+z})\ge 24+2.27.9=510.[/tex] Задачата е решена.
Върнете се в началото
Вижте профила на потребителя Изпратете лично съобщение
Titu_Andrescu
Напреднал


Регистриран на: 28 Oct 2006
Мнения: 370

Репутация: 68.9
гласове: 29

МнениеПуснато на: Sun Dec 30, 2007 4:51 pm    Заглавие:

Задача 52. Решение 2.

Имаме, че [tex](a^{\frac43}+b^{\frac43}+c^{\frac43})^2-(a^{\frac43})^2=(b^{\frac43}+c^{\frac43})(a^{\frac43}+a^{\frac43}+b^{\frac43}+c^{\frac43})\ge 2b^{\frac23}c^{\frac23}.4a^{\frac{2}{3}}b^{\frac13}c^{\frac13}[/tex] от неравенството между средно аритметично и средно геометрично.
Следователно [tex](a^{\frac43}+b^{\frac43}+c^{\frac43})^2\ge (a^{\frac43})^2+8a^{\frac23}bc=a^{\frac23}(a^2+bc)[/tex], което е еквивалентно на

[tex]\frac{a}{\sqrt{a^2+8bc}}\ge \frac{a^{\frac43}}{a^{\frac43}+b^{\frac43}+c^{\frac43}}[/tex].
Върнете се в началото
Вижте профила на потребителя Изпратете лично съобщение
Titu_Andrescu
Напреднал


Регистриран на: 28 Oct 2006
Мнения: 370

Репутация: 68.9
гласове: 29

МнениеПуснато на: Sun Dec 30, 2007 4:59 pm    Заглавие:

Задача 52. Решение 3

Полагаме [tex]x=\frac{a}{a+b+c},y=\frac{b}{a+b+c},z=\frac{c}{a+b+c}[/tex].
Неравенството може да се презапише в следния вид - [tex]xf(x^2+8yz)+yf(y^2+8zx)+zf(z^2+xy)\ge 1[/tex], където [tex]f(t)=\frac{1}{\sqrt{t}}[/tex].

Тъй като функцията [tex]f[/tex] e изпъкнала надолу в [tex]\mathbb{R^+}[/tex] и [tex]x+y+z=1[/tex], то можем да приложим неравенството на Дженсън.

[tex]xf(x^2+8yz)+yf(y^2+8zx)+zf(z^2+8xy)\ge f(x(x^2+8yz)+y(y^2+8zx)+z(z^2+8xy))[/tex].

Забележете, че [tex]f(1)=1[/tex] и тъй като функцията [tex]f[/tex] е строго намаляваща, то е дастатъчно да докажем, че [tex]1\ge x(x^2+8yz)+y(y^2+8zx)+z(z^2+8xy)[/tex].

Като използваме, че [tex]x+y+z=1[/tex] преобразуваме последното неравенство така, че да стане хомогенно:

[tex](x+y+z)^3\ge x(x^2+8yz)+y(y^2+8zx)+z(z^2+8xy)[/tex], което е вярно, защото
[tex](x+y+z)^3-x(x^2+8yz)-y(y^2+8zx)-z(z^2+8xy)=3\left[x(y-z)^2+y(z-x)^2+z(x-y)^2\right]\ge 0[/tex].
Върнете се в началото
Вижте профила на потребителя Изпратете лично съобщение
Мирослав Стоенчев
Напреднал


Регистриран на: 21 Aug 2007
Мнения: 279

Репутация: 72
гласове: 45

МнениеПуснато на: Sun Dec 30, 2007 5:55 pm    Заглавие:

Задача 44. Лема. Ако [tex]x,y\in R^{+},[/tex] то [tex]\sqrt[3]{\frac{x^{3}+y^{3}}{2}}\le \frac{x^{2}+y^{2}}{x+y}.[/tex]
Д-во:Полагаме [tex]\frac{x}{y}=t.[/tex] Тогава [tex]\sqrt[3]{\frac{x^{3}+y^{3}}{2}}\le \frac{x^{2}+y^{2}}{x+y}\Leftrightarrow (x^{3}+y^{3})(x+y)^{3}\le 2(x^{2}+y^{2})^{3}\Leftrightarrow (t^{3}+1)(t+1)^{3}\le 2(t^{2}+1)^{2}\Leftrightarrow t^{6}-3t^{5}+3t^{4}-2t^{3}+3t^{2}-3t+1\ge0.[/tex]
Разделяме на [tex]t^{3}[/tex] и полагаме [tex]t+\frac{1}{t}=u>0.[/tex] Имаме

[tex](t^{3}+\frac{1}{t^{3}})-3(t^{2}+\frac{1}{t^{2}})+3(t+\frac{1}{t})-2\ge0\Leftrightarrow (u^{3}-3u)-3(u^{2}-2)+3u-2\ge0\Leftrightarrow (u-2)^{2}(u+1)\ge 0,[/tex] което е изпълнено. Лемата е доказана.
Върнете се в началото
Вижте профила на потребителя Изпратете лично съобщение
Fed
VIP


Регистриран на: 24 May 2007
Мнения: 1136
Местожителство: София (Русе)
Репутация: 113.3
гласове: 33

МнениеПуснато на: Sun Dec 30, 2007 8:55 pm    Заглавие:

Задача 51. Решение.

Ще докажем че [tex]\sin x^{\sin x} < \cos x^{\cos x} \,[/tex] за [tex]\,\,x \in \left( {0,\frac{{\pi}}{4}} \right);[/tex]

Нека [tex]\sin x^{\sin x} = f(x)[/tex] и [tex]\cos x^{\cos x} = g(x)[/tex];


[tex]\begin{array}{l} f(x)' = \left( {e^{\sin x\ln \sin x} } \right)' = e^{\sin x\ln \sin x}.\left( {\cos x.\ln \sin x + \sin x.\frac{1}{{\sin x}}.\cos x} \right) = e^{\sin x\ln \sin x}.\cos x(\ln \sin x + 1) \\ g(x)' = \left( {e^{\cos x\ln \cos x} } \right)' = e^{\cos x\ln\cos x} .\left( { - \sin x.\ln \cos x + \cos x.\frac{1}{{\cos x}}.( - \sin x)} \right) = -e^{\cos x\ln \cos x} .\sin x(\ln \cos x + 1) \\ \end{array}[/tex]


Образуваме [tex]f(x)' = 0[/tex]. Търсим решение в [tex](0,\frac{{\pi}}{4})[/tex]
Оттук намираме [tex]x = \arcsin \frac{1}{e} \in \left( {0,\frac{{\pi}}{4}} \right)[/tex]

При [tex]x \in \left( {0,\arcsin \frac{1}{e}} \right) \Rightarrow \sin x < e^{ - 1} \Rightarrow \ln \sin x < - 1 \Rightarrow \ln \sin x + 1 < 0 \Rightarrow f(x)' < 0 \Rightarrow f(x) \downarrow [/tex]

Аналогично при [tex]x \in \left( {\arcsin \frac{1}{e},\frac{{\pi}}{4}} \right) \Rightarrow f(x) \uparrow [/tex]

Образуваме [tex]g(x)' = 0[/tex]. Търсим решение в [tex](0,\frac{{\pi}}{4})[/tex]
Намираме [tex]x = \arccos \frac{1}{e} \notin \left( {0,\frac{{\pi}}{4}} \right)[/tex]
Намираме [tex]g(x)'<0[/tex] в [tex](0,\frac{{\pi}}{4})[/tex]. Следователно [tex]g(x)[/tex] - монотонно намаляваща в този интервал.


[tex]f(0) = g(0);\,f(\frac{{\pi}}{4}) = g(\frac{{\pi}}{4});[/tex]

[tex]f(\arcsin \frac{1}{e}) = \sin (\arcsin \frac{1}{e})^{\sin (\arcsin \frac{1}{e})} = \left( {\frac{1}{e}} \right)^{\frac{1}{e}} \approx 0,692[/tex]

[tex]g(\arcsin \frac{1}{e}) = \cos (\arcsin \frac{1}{e})^{\cos (\arcsin \frac{1}{e})} = \sqrt {1 - \left( {\frac{1}{e}} \right)^2 } ^{\sqrt {1 - \left( {\frac{1}{e}} \right)^2 } } \approx 0,934[/tex]

За да направим окончателно заключението че [tex]\sin x^{\sin x} < \cos x^{\cos x}[/tex] използваме, че [tex]f(\arcsin \frac{1}{e}) < g(\arcsin \frac{1}{e})[/tex] и факта че уравнението [tex]\sin x^{\sin x} = \cos x^{\cos x}[/tex] няма решение в интервала [tex](0,\frac{{\pi}}{4})[/tex].

С това задачата е решена.
Върнете се в началото
Вижте профила на потребителя Изпратете лично съобщение
Мирослав Стоенчев
Напреднал


Регистриран на: 21 Aug 2007
Мнения: 279

Репутация: 72
гласове: 45

МнениеПуснато на: Mon Dec 31, 2007 12:22 am    Заглавие:

Задача 50. Ще докажем, че [tex]\sum_{c}\frac{m_{a}m_{b}}{ab}\ge \frac{9}{4}.[/tex] Неравенството е еквивалентно на [tex]\sum_{c}\sqrt{\left\[\frac{2(a^{2}+b^{2}+c^{2})}{a^{2}}-3\right\]\left\[\frac{2(a^{2}+b^{2}+c^{2})}{b^{2}}-3\right\]}\ge 9.[/tex]
Полагаме [tex]x=\frac{2(a^{2}+b^{2}+c^{2})}{a^{2}};y=\frac{2(a^{2}+b^{2}+c^{2})}{b^{2}};z=\frac{2(a^{2}+b^{2}+c^{2})}{c^{2}},[/tex] тогава неравенството добива вида

[tex]\sum_{c}\sqrt{(x-3)(y-3)}\ge 9\Leftrightarrow \sum_{c}(x-3)(y-3)+2\sqrt{(x-3)(y-3)(z-3)}\sum_{c}\sqrt{(x-3)}\ge 81.[/tex]

Полагаме [tex]\sigma_{1}=x+y+z;\sigma_{2}=xy+yz+zx;\sigma_{3}=xyz\Rightarrow \frac{\sigma_{2}}{\sigma_{3}}=\frac{1}{x}+\frac{1}{y}+\frac{1}{z}=\sum_{c}\frac{a^{2}}{2(a^{2}+b^{2}+c^{2})}=\frac{1}{2}\Rightarrow \sigma_{3}=2\sigma_{2}.[/tex] Пресмятаме [tex]\sum_{c}(x-3)(y-3)=\sigma_{2}-6\sigma_{1}+27; (x-3)(y-3)(z-3)=\sigma_{3}-3\sigma_{2}+9\sigma_{1}-27=9\sigma_{1}-\sigma_{2}-27.[/tex] Ще докажем, че

[tex]i)[/tex] [tex]\sigma_{2}-6\sigma_{1}\ge 0[/tex]

[tex]ii)[/tex] [tex]9\sigma_{1}-\sigma_{2}\ge 54.[/tex]

Имаме [tex]\sigma_{2}-6\sigma_{1}\ge 0\Leftrightarrow (a^{2}-b^{2})^{2}+(b^{2}-c^{2})^{2}+(c^{2}-a^{2})^{2}\ge 0,[/tex] също така

[tex]9\sigma_{1}-\sigma_{2}\ge 54\Leftrightarrow 9(a^{2}+b^{2}+c^{2})(a^{2}b^{2}+b^{2}c^{2}+c^{2}a^{2})-2(a^{2}+b^{2}+c^{2})^{3}-27(abc)^{2}\ge0.[/tex] Полагаме [tex]a=p+q;b=q+r;c=r+p[/tex] и замествайки в последното неравенство, се убеждаваме, че е изпълнено. Така [tex]ii)[/tex] е доказано. Използвайки [tex]i),ii)[/tex] заключаваме, че [tex]\sum_{c}(x-3)(y-3)=\sigma_{2}-6\sigma_{1}+27\ge 27;(x-3)(y-3)(z-3)=9\sigma_{1}-\sigma_{2}-27\ge 27.[/tex] Следователно [tex]\sum_{c}\sqrt{(x-3)}=\sqrt{(x-3)}+\sqrt{(y-3)}+\sqrt{(z-3)}\ge \sqrt[6]{(x-3)(y-3)(z-3)}\ge 3\sqrt{3}.[/tex] Окончателно

[tex]\sum_{c}(x-3)(y-3)+2\sqrt{(x-3)(y-3)(z-3)}\sum_{c}\sqrt{(x-3)}=\sigma_{2}-6\sigma_{1}+27+2\sqrt{9\sigma_{1}-\sigma_{2}-27}\sum_{c}\sqrt{(x-3)}\ge 27+2.(3\sqrt{3})(3\sqrt{3})=81.[/tex] Неравенството е доказано.

Решението е вярно само за остроъгълен триъгълник.
Върнете се в началото
Вижте профила на потребителя Изпратете лично съобщение
Titu_Andrescu
Напреднал


Регистриран на: 28 Oct 2006
Мнения: 370

Репутация: 68.9
гласове: 29

МнениеПуснато на: Mon Dec 31, 2007 3:58 pm    Заглавие:

Задача 50.

Лема. Ако [tex]a,b,c[/tex] са страните на даден триъгълник, [tex]x,y,z[/tex] са реални числа и [tex]M[/tex] е напълно произволна точка. Тогава
[tex](x+y+z)(xMA^2+yMB^2+zMC^2)\ge yza^2+zxb^2+xyc^2[/tex].
(тази лема е изключително силна)

Д-во: Най-лесното доказателство на тази мощна лема използва вектори. Очевидно имаме [tex]\left(x.\vec{MA} +y\vec{MB} +z\vec{MC} \right)^2\ge 0[/tex].
С други думи, [tex]x^2\vec{MA}^2+y^2\vec{MB}^2+z^2\vec{MC}^2+2xy\vec{MA}\vec{MB}+2yz\vec{MB} \vec{MC} +2zx\vec{MC} \vec{MA}\ge 0 [/tex].
Тъй като квадрата на даден вектор е равен на квадрата на дължината му, то можем да запишем горното неравенство по следния начин:
[tex]x^2{MA}^2+y^2{MB}^2+z^2{MC}^2+2xy\vec{MA}\vec{MB}+2yz\vec{MB} \vec{MC} +2zx\vec{MC} \vec{MA}\ge 0 [/tex] (~).

Сега правим така, [tex]2\vec{MB} \vec{MC} =\vec{MB} ^2+\vec{MC}^2-(\vec{MB} ^2+\vec{MC}^2-2\vec{MB} \vec{MC} ) =\vec{MB}^2+\vec{MC} ^2-(\vec{MB}-\vec{MC} )^2 =\vec{MB} ^2+\vec{MC} ^2-\vec{BC} ^2=MB^2+MC^2-a^2[/tex].
Аналогично доказваме, че [tex]2\vec{MA} \vec{MB} =MA^2+MB^2-c^2[/tex] и [tex]2\vec{MC} \vec{MA} =MC^2+MA^2-b^2[/tex].

Като заместим в (~) получаваме, че
[tex]x^2{MA}^2+y^2{MB}^2+z^2{MC}^2+xy(MA^2+MB^2-c^2)+yz(MB^2+MC^2-a^2) +zx(MC^2+MA^2-b^2)\ge 0 [/tex],
което лесно се свежда до [tex](x+y+z)(xMA^2+yMB^2+zMC^2)\ge yza^2+zxb^2+xyc^2[/tex].
С това лемата е доказана.


Полагаме [tex]x=\frac{a}{m_a},y=\frac{b}{m_b},z=\frac{c}{m_c}[/tex] и слагаме точката [tex]M[/tex] да бъде медицентъра на триъгълника, тогава
[tex]\left(\frac{a}{m_a}+\frac{b}{m_b}+\frac{c}{m_c}\right).\frac49.(am_a+bm_b+cm_c)\ge \frac{abc}{m_am_bm_c}(am_a+bm_b+cm_c)[/tex], което е еквивалентно на [tex]\frac{m_am_b}{ab}+\frac{m_bm_c}{bc}+\frac{m_cm_a}{ca}\ge \frac94[/tex].
Върнете се в началото
Вижте профила на потребителя Изпратете лично съобщение
Titu_Andrescu
Напреднал


Регистриран на: 28 Oct 2006
Мнения: 370

Репутация: 68.9
гласове: 29

МнениеПуснато на: Mon Dec 31, 2007 4:18 pm    Заглавие:

Задача 53. Да се докаже, че [tex]\sum_{i=1}^{n}\frac{x_i}{x_{i+1}+x_{i+2}}>(\sqrt{2}-1)n[/tex], където [tex]x_{n+1}=x_1,x_{n+2}=x_2[/tex] и [tex]x_1,x_2,...,x_n>0[/tex].

(Молдова 2005 г., обобщеното неравенство на Несбит)
Върнете се в началото
Вижте профила на потребителя Изпратете лично съобщение
LogiC
Начинаещ


Регистриран на: 28 Dec 2007
Мнения: 3
Местожителство: Sofia / Simeonovo
Репутация: 1.3
гласове: 1

МнениеПуснато на: Mon Dec 31, 2007 7:49 pm    Заглавие:

(IMO 2001 г.)
Това книжка ли е ? Smile И ако е , дайте линк Smile
Върнете се в началото
Вижте профила на потребителя Изпратете лично съобщение Посетете сайта на потребителя
Titu_Andrescu
Напреднал


Регистриран на: 28 Oct 2006
Мнения: 370

Репутация: 68.9
гласове: 29

МнениеПуснато на: Tue Jan 01, 2008 3:44 am    Заглавие:

IMO - Internatioanal Mathematical Olympiad, Международна Олимпиада по Математика
Върнете се в началото
Вижте профила на потребителя Изпратете лично съобщение
Покажи мнения от преди:   
   Форум за математика Форуми -> Олимпиади и състезания за 9-12 клас Часовете са според зоната GMT + 2 Часа
Иди на страница Предишна  1, 2, 3, 4, 5, 6, 7, 8  Следваща
Страница 5 от 8

 
Идете на:  
Не Можете да пускате нови теми
Не Можете да отговаряте на темите
Не Можете да променяте съобщенията си
Не Можете да изтривате съобщенията си
Не Можете да гласувате в анкети
Може да прикачвате файлове
Може да сваляте файлове от този форум
Copyright © 2005-2021 math10.com.